Une petite somme :-) — Les-mathematiques.net The most powerful custom community solution in the world

Une petite somme :-)

Bonjour, je me suis demandé combien vaut $\displaystyle \sum_{n=1}^{\infty} \frac{e^{-n\pi}}{n\pi}.$

J'ai donc eu une idée : on a
$$\sum_{k=1}^{\infty} x^k = \frac{x}{1-x},
$$ d'où $\ \displaystyle \sum_{n=1}^{\infty} e^{-n\pi}=\frac{e^{-\pi}}{1-e^{-\pi}},$
après on définit une fonction : $\displaystyle \lambda(x)= \frac{e^{-\pi x}}{1-e^{-\pi x}}= \sum_{n=1}^{\infty} e^{-n\pi x},$
et du coup on a $\displaystyle \int_{1}^{\infty} \lambda(x)dx = \sum_{n=1}^{\infty} \frac{e^{-n\pi}}{n\pi}.$

Maintenant calculons l'intégrale.
$\displaystyle I= \int_{1}^{\infty} \frac{e^{-\pi x}}{1-e^{-\pi x}}dx.$ Si on pose $\displaystyle y=e^{-\pi x}; -\frac{dy}{\pi}e^{\pi x}=dx$
$\displaystyle I=\frac{1}{\pi} \int_{0}^{e^{-\pi}} \frac{1}{1-y}dy.$
Si on pose $t=1-y,$ on a $dt=-dy$
$\displaystyle I=\frac{1}{\pi} \int_{1-e^{-\pi}}^{1} \frac{1}{t}dt=\frac{1}{\pi} \ln(1)- \frac{1}{\pi}\ln(1-e^{-\pi})= \frac{1}{\pi} \ln \big( \frac{1}{1-e^{-\pi}}\big).$
Donc on a : $\displaystyle \sum_{n=1}^{\infty} \frac{e^{-n\pi}}{n\pi}=\frac{1}{\pi} \ln \big( \frac{1}{1-e^{-\pi}}\big).$
Mais... ça me semble trop simple pour être vrai :) et je ne trouve pas l'erreur... ._.' Qui veut bien m'aider à la trouver ? :-)

[Ne pas confondre la préposition 'à' avec 'a' (3ème personne du présent de l'indicatif du verbe avoir).
Pour distinguer : si on peut remplacer par 'avait', c'est le 'a' du verbe avoir, sinon c'est la préposition 'à'. AD]


[Merci beaucoup :]
Je suis donc je pense 

Réponses

  • On peut aussi écrire: $$\sum_{n=1}^{\infty} \frac{e^{-n\pi}}{n}=\ln \big(\frac{1}{1-e^{-\pi}} \big)$$
    Je suis donc je pense 
  • Je n'ai pas vérifié tous tes calculs, mais c'est la bonne idée, on reconnaît une primitive d'une série géométrique, et des théorèmes usuels justifient qu'on a bien le droit de primitiver terme à terme dans la série.
  • Bonjour,

    Votre méthode me paraît correcte (bon, à quelques endroits il pourrait être utile d'écrire $x \in ...$ pour éviter de possibles étourderies), mais elle justifie une interversion série/intégrale (pas bien dure à justifier, on peut utiliser des séries entières). Une autre possibilité (qui utilise l'interversion dérivation/série dans le cas de séries entières), qui revient presque au même mais en évitant de s'embêter avec des exponentielles:

    On peut définir, pour $x$ réel de valeur absolue strictement inférieure à 1, $\displaystyle f(x) = \sum\limits_{k = 1}^{+ \infty} \frac{x^k}{k}$.

    On dérive alors $f$ (légitimement, comme fonction somme d'une série entière de rayon de converge égal à 1): $\displaystyle \forall x \in ]-1, 1[, f'(x) = \sum\limits_{k = 1}^{+ \infty} x^{k - 1} = \frac{1}{1 - x}$.

    Mais une primitive de $x \mapsto \frac{1}{1 - x}$ sur $]-1, 1[$ est $x \mapsto - \ln(1 - x)$, qui s'annule en 0 comme $f$.

    Donc $\forall x \in ]-1, 1[$, $\displaystyle \sum\limits_{k = 1}^{+ \infty} \frac{x^k}{k} = f(x) = - \ln(1 - x)$. Il suffit essentiellement de calculer $f(e^{-\pi})$ pour trouver ce que vous avez trouvé.

    Mais bon, quitte à connaître les séries entières, autant savoir directement que $\forall x \in [-1, 1[, \displaystyle \sum\limits_{k = 1}^{+ \infty} \frac{x^k}{k} = - \ln(1 - x)$
  • Le développement en série entière, $\displaystyle \sum_{n=1}^\infty \dfrac{x^n}{n}$ est bien connu pour être égal à $-\ln(1-x)$ quand $-1\leq x<1$.

    Or, $-1\leq \text{e}^{-\pi}<1$ donc on a bien que $\displaystyle \sum_{n=1}^\infty \dfrac{\text{e}^{-\pi n}}{n}=-\ln\left(1-\text{e}^{-\pi}\right)$
  • Oui, j'aurais du y penser
    Je suis donc je pense 
  • On peut démontrer ce résultat avec peu d'outils.
    \begin{align}
    x\neq 1,\quad& \sum_{n=0}^N x^n=\dfrac{1-x^{N+1}}{1-x}, \\
    \text{donc :}& \\
    x\neq 1,\quad &\dfrac{1}{1-x}=\sum_{n=0}^N x^n+\dfrac{x^{N+1}}{1-x}.
    \end{align} On intègre l'égalité entre $0$ et $0<y<1$ :
    \begin{align}-\ln(1-y)&=\sum_{n=0}^{N}\frac{y^{n+1}}{n+1}+\int_0^y \frac{x^{N+1}}{1-x}dx\\
    &=\sum_{n=1}^{N+1}\frac{y^{n}}{n}+\int_0^y \frac{x^{N+1}}{1-x}dx.

    \end{align} L'intégrale tend vers $0$ quand $N$ tend vers l'infini, à $0<y<1$ fixé.
    La fonction $x\mapsto \dfrac{1}{1-x}$ est croissante et positive, sur l'intervalle $[0,y],\ 0<y<1$ on peut la majorer par $\dfrac{1}{1-y}$ donc :
    \begin{align}0<\int_0^y \frac{x^{N+1}}{1-x}dx<\frac{1}{1-y}\frac{y^{N+2}}{N+2}.
    \end{align} Le dernier terme tend vers $0$ quand $N$ tend l'infini, à $0<y<1$ fixé.
Connectez-vous ou Inscrivez-vous pour répondre.
Success message!